LSAT and Law School Admissions Forum

Get expert LSAT preparation and law school admissions advice from PowerScore Test Preparation.

 Administrator
PowerScore Staff
  • PowerScore Staff
  • Posts: 8916
  • Joined: Feb 02, 2011
|
#26192
Complete Question Explanation

Assumption—CE. The correct answer choice is (C)

This question illustrates the use of a Supporter Assumption to connect a logical gap in a causal argument. The conclusion here is that cancer-patient support groups may have therapeutic value. This is based on two observations: (1) support groups reduce participants’ stress levels, i.e. they cause stress levels to go down; and (2) a weakened immune system increases vulnerability to cancer, i.e. it causes an increase in vulnerability to cancer.

  • ..... ..... Cause ..... ..... ..... ..... ..... ..... ..... Effect

    Premise: ..... Support groups ..... ..... :arrow: ..... Reduce stress levels

    Premise: ..... Weakened immune systems ..... :arrow: ..... Increase vulnerability to cancer

    Conclusion: ..... Support groups ..... ..... :arrow: ..... Therapeutic value (i.e. reduced vulnerability to cancer)

The author never explicitly connects the support groups and their stress reducing effects to a weakened immune system. Why would reducing stress have any therapeutic value, unless there were a connection between stress and the immune system? If the author believes the support groups are therapeutic, then he must assume that stress can weaken the immune system. This prephrase reveals answer choice (C) to be correct.

Answer choice (A): This answer choice is incorrect because it does not have to be true in order for the conclusion to follow. Cancer patients do not need the ability to function well under extreme stress. In fact, extreme stress was never a relevant consideration in this argument.

If you apply the Assumption Negation Technique to this answer choice, the statement becomes “cancer patients cannot learn to function well under extreme stress.” If patients lack this ability, it has no effect on whether or not support groups can be therapeutic. Since the negated answer choice does not weaken the conclusion of the argument, the answer is incorrect.

Answer choice (B): This answer choice strengthens the argument, but is not an assumption upon which the conclusion depends. The Assumption Negation Technique easily eliminates it: even if disease were a biochemical phenomenon, cancer-patient support groups could still be therapeutic. Since the negated answer choice has no effect on the argument, the answer is not an assumption of the argument.

Answer Choice (C): This is the correct answer choice. This answer choice suggests stress as a potential cause of a weakened immune system. As stated above, the author must assume a connection between these two ideas for the conclusion about support groups to be valid. If, by reducing stress, support groups may have therapeutic value, then there must be a causal connection between stress and a weakened immune system.


Answer choice (D): This answer choice is tempting but incorrect. The author already stated that support groups reduce stress in the premises. While this answer choice may help explain why these groups reduce stress, the answer choice is not necessary for the conclusion to be true. In other words, discussing one’s condition does not have to eliminate stress in order for these support groups to have therapeutic value. This answer strengthens the argument by making it more likely that support groups reduce stress, is not necessary for the argument to be true.

Answer choice (E): This answer choice actually weakens the argument. Rather than showing that stress causes a weakened immune system, this answer choice suggests that stress is itself an effect of a weakened immune system. If stress is merely a symptom of a weakened immune system, then reducing stress is unlikely to have an effect on one’s immune system, which calls into question the therapeutic value of support meetings.
 sodomojo
  • Posts: 24
  • Joined: Aug 01, 2017
|
#40126
I'm misunderstanding the argument here (kind of annoying for what seems to be an easy question).

Am I wrong to assume that "cancer-patient support groups" refers to support groups for patients who already have cancer? What's the point of reducing stress levels in an attempt to lower their vulnerability to cancer, if they already have cancer?

I picked (E) on the assumption that weakened immune systems led to the participants' cancer and stress levels, and so the therapeutic value came in the form of minimizing the latter symptom. Where did I go wrong here?
User avatar
 Dave Killoran
PowerScore Staff
  • PowerScore Staff
  • Posts: 5853
  • Joined: Mar 25, 2011
|
#40129
Hi Sodomojo,

Thanks for the question! First off, I really appreciate the explanation you provided on how you analyzed this problem. It makes it much easier for us to see what you were doing and where you might have run into problems. So, thanks very much for that! That said, let's look at what happened here.

You were not wrong to assume that "cancer-patient support groups" refers to support groups for patients who already have cancer. That's exactly what they meant, so the problem doesn't come from that part. Instead, it's what you say afterwards, that "What's the point of reducing stress levels in an attempt to lower their vulnerability to cancer, if they already have cancer?" I get the sense that you are looking at having cancer as a sort of binary yes/no condition, where you either have it or you don't, and that's the main (or only) consideration. While it is true that you either have it or you don't, that isn't the key here; instead, the key is the "vulnerability" aspect. While two people may both have cancer, one might be in a much stronger position, and far less likely to die from the disease than another person, who might be more vulnerable to the negative effects of the disease. So, the argument here is about how stress and a weakened immune system can affect the strength of the disease once you have it. I suspect that seeing it from that angle will probably unlock this problem for you, but I can see exactly what you were thinking about this being more about whether you have cancer or not.

Interestingly, (C) and (E) are very similar answers: whereas (C) says that a weakened immune system can be caused by stress, (E) says that stress is caused by a weakened immune system. So, each answer is the reverse of the other, and you were really close in choosing (E). Answer choice (C) ends up being correct because in the argument the author is connecting cancer-support groups to lower stress, and then saying that benefits cancer patients because a weakened immune system increase vulnerability, so there has to be a connection between stress and weakened immune systems, and the assumption involves a chain that includes the idea that stress can weaken the immune system.

Please let me know if that helps. Thanks!
 sodomojo
  • Posts: 24
  • Joined: Aug 01, 2017
|
#40172
Ah - my inability to keep an flexible mind strikes again.

Thanks so much for the thorough explanation Dave!

Get the most out of your LSAT Prep Plus subscription.

Analyze and track your performance with our Testing and Analytics Package.